You are on page 1of 11

PSET2

Subham Goenka
May 2022

1 Question 1

we see that ∠CAZ = 90−


B
and ∠CDZ = 90 + B
since their sum is 180 AZDC is cyclic

1
2 Question 2

we can extend CM to meet AD at H


in ∆DHQ
sin ∠HDQ sin ∠DHQ
=
HQ DQ
and in ∆CQD
sin ∠QDC sin ∠DCQ
=
CQ DQ
and
∠DCQ = ∠AM H
sin ∠AHM AH AH HQ
= = =
sin ∠AM H AM CN QC
combining the equations we get
sin ∠HDQ = sin ∠QDC
=⇒ ∠HDQ = ∠QDC
hence proved

3 Question 3
we can divide this set into 3 sets-
S1 = {2, 6, 10, 14, 18}
S2 = {4, 8, 12, 16, 20}
S3 = {1, 3, 5, 7, . . . , 17, 19} we can calculate the number subsets not divisible by
4 that is either we can choose 3 elements from S3 or 2 elements from S3 and 1
element from S1 i.e    
10 10
+ ∗5
3 2

2
so total number of subsets whose product is divisble by 4 is
     
20 10 10
− − ∗ 5 = 795
3 3 2

4 Question 4
WLOG assume p ≤ q
we can take
k1 p = 30q − 1 (1)
and
k2 q = 30p − 1 (2)
so
k2 q = 30p − 1 ≤ 30q − 1
which implies k2 < 30
clearly k2 does not share a divisor with 30 except 1
so k2 = 1, 7, 11, 13, 17, 19, 23, 29
 
30p − 1
k1 p = 30 −1
k2

=⇒ (900 − k1 k2 )p = k2 + 30
similarly
(900 − k1 k2 )q = k1 + 30

k2 + 30 k1 + 30
=⇒ =
p q
k2 + 30 is prime for all values of k2 except 19
case 1:
k2 ̸= 19
so p = k2 + 30
k2 (q − 30) = 31 ∗ 29
clearly k2 = 1, 29 which gives (p, q) = (31, 929), (59, 61)

case 2: k2 = 19
we get (p, q) = (7, 11)
so (p, q) = (31, 929), (59, 61), (7, 11) are all the solutions

3
5 Question 5
lets assume there exists an i such that ai ≥ a + 1
so
ai−1 + ai+1
a + 1 ≤ ai ≤
2
clearly either ai−1 or ai+1 is greater than or equal to a+1
assume ai+1 ≥ a + 1

claim: ak+1 ≥ ak for all k ≥ i


proof:It is clearly true for k = i,
we can assume aj ≤ aj+1
aj + aj+2
aj+1 ≤
2
it follows that aj+1 ≤ aj+2
so a + 1 ≤ ai ≤ ai+1 ≤ ai+2 · · · ≤ an

∴ contradiction

6 Question 6
lets assume such arrangement is possible then let the number of queens in the
quadrant(2) adjacent to both the quadrants(1 and 3) where queens can be placed
be y and the number of queens in other two quadrants be x and z so

x + y + z = 100

in quadrant 1 and 2 atmost 50 queens can be placed as their are 50 rows

x + y ≤ 50

similarly
y + z ≤ 50
x + 2y + z ≤ 100
y=0
so number of queens in quadrant 2 is 0
In quadrant 1 and 3 together there are 99 diagnols in the same direction but
there are 100 queens, so by pigeonhole principle one diagnol contains 2 queens.
∴ contradiction

4
7 Question 7
we know that √ √ √
n − 1 < ⌊ n⌋ ≤ n
there exists a k such that
√ √
k + 1 ≤ ⌊ n⌋(⌊ n⌋ + k) = n
√ √ √
n + (k − 2) n + 1 − k < ⌊ n⌋(⌊ n⌋ + k) = n
for k ≤ 2 the above inequality holds true
√ √ √
⌊ n⌋(⌊ n⌋ + k) = n < (⌊ n⌋ + 1)2

so values of n of the form a2 , a(a + 1), a(a + 2) where a ∈ N satisfy the condition
for 4 ≤ k
√ √ √ √ √
n < n + k k + 1 + 1 − k ≤ n + (k − 2) n + 1 − k < ⌊ n⌋(⌊ n⌋ + k) = n
not possible
for k = 3

n+ n−2<n
n<4
we get no new solution
so values of n of the form a2 , a(a + 1), a(a + 2) where a ∈ N are all the solutions

8 Question 8
We can see that√left hand
√ side is a strictly increasing function
let f (x) = n + ⌊ n⌋ + ⌊ 3 n⌋

f (1936) < 2022


so 1936 < n < 2022
√ √
⌊ n⌋ = 44, ⌊ 3 n⌋ = 12
so we get n = 1966

5
9 Question 9
for the sake of contradiction assume a + b + c + d = 0

abc + bcd + cda + dab = a + b + c + d + 1 + 2 + 3 − 6


=⇒ abc + bcd + cda + dab = 0
consider a quintic polynomial with roots a, b, c, d

P (x) = x4 + lx3 + mx2 + nx + c

l = 0, n = 0
let k = x2 we can see that k can have two possible values
so a, b, c, d are made up of two pairs where in a pair one element is the negative
of the other
case 1:
(a, b, c, d) = (p, −p, q, −q)
pq 2 − p = 2
−pq 2 + p = 3
case 2:
(a, b, c, d) = (p, q, −p, −q)
−p2 q + q = 1
p2 q − q = 3
case 3:
(a, b, c, d) = (p, q, −q, p)
−pq 2 + p = 1
pq 2 − p = 2
In all three cases we achieve contradiction

10 Question 10
we know that
x2 ≡ 0, 1, 4 mod 8
so if x is a prime
x2 ≡ 1 mod 8
except x = 2
let the first l terms be 2 then

4l + 12 − l ≡ 1 mod 8
3l + 12 ≡ 1 mod 8

6
so l = 7

let the number of 3s be k


we know that

p2 ≡ 1 mod 6
for all primes except 2 and 3 so

28 + 3k + 5 − k ≡ 1 mod 6
2k + 3 ≡ 1 mod 6
clearly k ̸= 5
so k = 2

one element is 7 as 2p1 + p9 = 7


case 1:
(p10 , p11 ) = (5, 5)
we see that p12 = 7
this has no solution

case 2: (p10 , p11 ) = (5, 7)

120 = (p13 + p12 )(p13 − p12 )


we get (p12 , p13 ) = (7, 13), (13, 17), (29, 31)

case 3: p10 = 7
we know that
p2 ≡ 1, 4 mod 5
for a prime p

95 + p211 + p212 = p213

taking mod 5 we can see that there are no solution

so (2,2,2,2,2,2,2,3,3,5,7,7,13), (2,2,2,2,2,2,2,3,3,5,7,13,17), (2,2,2,2,2,2,2,3,3,5,7,29,31)


are the only solutions

11 Question 11
we can see by a simple calculation
(n+1
2 )

2 n+1
(n+1  = n−3
2 )
2

7
let a3 , a4 , . . . , an be a sequence replicating the original sequence

a3 = 3

and
n+1
an = an−1
n−3
so
nan − (n + 4)an−1 = 3an − 3an−1
(n − 1)an−1 − (n + 3)an−2 = 3an−1 − 3an−2
..
.
4a4 − 8a3 = 3a4 − 3a3
adding up we get
n−1
(n − 3)an X
= ak
5
k=3

substituting n=41 we get


38 ∗ a41
≡4 mod 1000
5

8
12 Question 12

let ∠BAZ = β and ∠CAZ = α


through some angle chasing we get
α−β
∠BZX =
2
and
α−β
∠Y ZC =
2
as ∠AXZ = 90 − β we get ∠OZY = β
we also get ∠AZO = α−β 2
it is easy to see that ∠BZO = ∠CZO
hence proved

13 Question 13
clearly m? is divisible by 2 but not by 4 so

m≡2 mod 4

n(n + 1)(n + 2)(n + 3) ≡ 0 mod 4


hence there is no solution

9
14 Question 14

using sine rule in ∆ABD we get that


   
δ δ
BD cos = AD cos β −
2 2

similarly in ∆ABC
   
δ δ
BC cos = AC sin β −
2 2
using cosine rule in ∆BCD and ∆ACD
we get

BD2 + BC 2 − 2BCBD cos(δ) = AC 2 + AD2 − 2ACAD sin(2β − δ)


  
2 2 2 2 δ δ
=⇒ BD +BC −2BCBD cos(δ) = AC +AD −4ACAD sin β − β−
2 2
δ
=⇒ BD2 + BC 2 + 2BCBD(2 cos2 ( ) − cos(δ)) = AC 2 + AD2
2
=⇒ BD2 + BC 2 + 2BCBD = AC 2 + AD2
=⇒ (BC + BD)2 = AC 2 + AD2

10
15 Question 15
sum of all weights needs to be divisible by 3 so
n(n + 1)
≡0 mod 3
2
n is of the form 3k or 3k + 2
case 1: n = 3k
for n = 6
 
1 2 3
6 5 4
works suppose it works for some n = 3k
where 1,2 and 3 are in seperate groups

we can use the following algorithm-

add 3k+3 in group which contains 3


add 3k+1 in group which contains 2
add 3k+2 in group which contains 1
swap 2 and 3 and we are done
case 2:n = 3k + 2
for n = 5
 
1 2 5
4 3
works

suppose it works for somen = 3k + 2


we can use the following algorithm-

add 3k + 5 to the group with 1


add 3k + 4 to the group with 2
add 3k + 3 to the group which has neither of those
now add 1 to the group of 3k + 3
and we are done.
so it works for all values of n of the form n = 3k, 3k + 2 except 3

16 Question 16
2x2 + y 2 + 7 = 2(x + 1)(y + 1) =⇒ 2x2 + y 2 − 2xy − 2x − 2y + 5 = 0

(y − x − 1)2 + (x − 2)2 = 0
so x = 2 and y = 3

11

You might also like